K
Khách

Hãy nhập câu hỏi của bạn vào đây, nếu là tài khoản VIP, bạn sẽ được ưu tiên trả lời.

11 tháng 1 2018

Mai giải cho

3 tháng 12 2018

\(\frac{1}{a^2}+\frac{1}{b^2}+\frac{1}{c^2}\ge a^2+b^2+c^2\)

<=>  \(\frac{1}{a^2}+\frac{1}{b^2}+\frac{1}{c^2}+2\left(ab+bc+ca\right)\ge a^2+b^2+c^2+2\left(ab+bc+ca\right)\)

<=>  \(\frac{1}{a^2}+\frac{1}{b^2}+\frac{1}{c^2}+2\left(ab+bc+ca\right)\ge9\)

Ap dung BDT AM-GM ta co:

\(\frac{1}{a^2}+\frac{1}{b^2}+\frac{1}{c^2}+2\left(ab+bc+ca\right)\ge\frac{1}{ab}+\frac{1}{bc}+\frac{1}{ca}+2\left(ab+bc+ca\right)\)

\(=\frac{3}{abc}+\left(ab+bc+ca\right)+\left(ab+bc+ca\right)\)

\(\ge3\sqrt[3]{\frac{3}{abc}\left(ab+bc+ca\right)\left(ab+bc+ca\right)}\)

\(\ge3\sqrt[3]{\frac{3}{abc}.3abc\left(a+b+c\right)}=9\)

=>  dpcm

5 tháng 7 2016

Các bạn cố gắng giúp mình nha . Mình xin chân thành cảm ơn